Đến nội dung

nhungvienkimcuong

nhungvienkimcuong

Đăng ký: 23-12-2014
Offline Đăng nhập: Riêng tư
****-

#744618 $\sum \frac{a^{2}+2ab}{(a-b)^{2...

Gửi bởi nhungvienkimcuong trong 21-04-2024 - 21:34

Cho a,b,c là các số thực đôi một phân biệt, CMR $\sum \frac{a^{2}+2ab}{(a-b)^{2}} \geq \frac{2}{3}$

Chú ý rằng $\frac{a^2+2ab}{(a-b)^2}+\frac{1}{3}=\frac{(2a+b)^2}{3(a-b)^2}$, do vậy ta cần chứng minh

\[\left ( \frac{2a+b}{a-b} \right )^2+\left ( \frac{2b+c}{b-c} \right )^2+\left ( \frac{2c+a}{c-a} \right )^2\ge 5.\]

Đặt $x=\frac{2a+b}{a-b},y=\frac{2b+c}{b-c},z= \frac{2c+a}{c-a}$ thì

\[(x+1)(y+1)(z+1)=(x-2)(y-2)(z-2)\implies xy+yz+zx+3=x+y+z.\]

Khi đó 

\begin{align*}x^2+y^2+z^2&=(x+y+z)^2-2(xy+yz+zx)\\ &=(x+y+z)^2-2(x+y+z-3)=(x+y+z-1)^2+5\ge 5.\end{align*}

Như vậy giải quyết bài toán hoàn toàn, dấu bằng xảy ra chẳng hạn khi $a=-1,b=2$ và $c=0$.

 

Ghi chú. Cách xử lí này nổi tiếng với bài IMO 2008 và một số bất đẳng thức Đào Hải Long.




#744607 $\sum_{n\vdots d,d=2k+1}\varphi (d)2^{...

Gửi bởi nhungvienkimcuong trong 20-04-2024 - 22:23

Cho $n$ là số nguyên dương. Chứng minh rằng

$$\sum_{n\vdots d,d=2k+1}\varphi (d)2^{\frac{n}{d}} \hspace{0.2cm} \vdots \hspace{0.2cm} n$$

Với $\varphi (x)$ là số các số nguyên dương nhỏ hơn hoặc bằng $x$ và nguyên tố cùng nhau với $x$.

Ta sẽ đếm số tập hợp $A$ là tập hợp con của $\{1,2,\dots,n\}$ sao cho tổng các phần tử của $A$ là bội của $n$.

 

Đặt $\epsilon=e^{2i\pi/n}$ là căn nguyên thủy bậc $n$ của đơn vị, khi đó ta có tính chất

\[\sum_{k=1}^{n}\epsilon^{km}=n\big[m\equiv 0\pmod{n}\big],\]

với $[P]$ là kí hiệu Iverson. Từ kết quả này suy ra ngay số tập hợp cần tính chính là

\[\mathcal{K}=\sum_{A\subset\{1,\dots,n\}}\frac{\sum_{k=1}^{n}\epsilon^{k\cdot s(A)}}{n}=\frac{1}{n}\sum_{A\subset\{1,\dots,n\}}\sum_{k=1}^{n}\epsilon^{k\cdot s(A)},\]

trong đó $s(A)$ tổng các phần tử của $A$. Tiếp theo biến đổi

\[\sum_{A\subset\{1,\dots,n\}}\sum_{k=1}^{n}\epsilon^{k\cdot s(A)}=\sum_{k=1}^{n}\sum_{A\subset\{1,\dots,n\}}\prod_{a\in A}\epsilon^{ka}=\sum_{k=1}^{n}\prod_{a=1}^n(1+\epsilon^{ka}).\]

Như vậy $\prod_{a=1}^n(1+\epsilon^{ka})=2^{\text{UCLN}(k,n)}\left[\frac{n}{\text{UCLN}(k,n)}\equiv 1\pmod{2}\right]$ nên ta có 

\[\mathcal{K}=\frac{1}{n}\sum_{k=1}^n2^{\text{UCLN}(k,n)}\left[\frac{n}{\text{UCLN}(k,n)}\equiv 1\pmod{2}\right].\]

Phần còn lại chỉ cần chứng minh đẳng thức (của bạn đọc)

\[\sum_{k=1}^n2^{\text{UCLN}(k,n)}\left[\frac{n}{\text{UCLN}(k,n)}\equiv 1\pmod{2}\right]=\sum_{d\mid n}\varphi (d)2^{\frac{n}{d}}\big[d\equiv 1\pmod{2}\big].\]

 

 

Ghi chú. Đẳng thức này khá liên quan với đếm Necklace (lời giải sơ cấp đã có ở diễn đàn).




#744604 $n^2 + m$ không là số chính phương với $m | 2n^2$

Gửi bởi nhungvienkimcuong trong 20-04-2024 - 20:15

Cho $n$ là số nguyên dương và $m$ là ước nguyên dương của $2n^2$ Chứng minh rằng $n^2 + m$ không là số chính phương

Theo giả thiết thì $a=\frac{2n^2}{m}$ là số nguyên dương, khi đó

\[a^2(n^2+m)=n^2(a^2+2a).\]

Vì $a^2<a^2+2a<(a+1)^2$ nên $a^2+2a$ không thể là số chính phương, từ đây ta có điều cần chứng minh.




#744378 Đề thi chọn đội tuyển Olympic quốc tế (TST) năm 2024

Gửi bởi nhungvienkimcuong trong 26-03-2024 - 16:31

Đề thi chọn đội tuyển Olympic quốc tế năm 2024

Thời gian: 270 phút

 

Ngày thi thứ nhất: 26/03/2024

 

Bài 1. Cho đa thức $P(x)$ hệ số thực, khác hằng và hệ số của bậc cao nhất là $1$. Tìm tất cả các hàm số $f\colon \mathbb{R}\to\mathbb{R}$ liên tục và thỏa mãn

\[f\Big(f\big(P(x)\big)+y+2023f(y) \Big)=P(x)+2024f(y)\]

với mọi $x,y\in \mathbb{R}$.

 

Bài 2. Một khu vườn có mặt bằng là lưới ô vuông $2024 \times 2024$. Người làm vườn đặt các chậu hoa thỏa mãn đồng thời các điều kiện:

  1. Một chậu trồng đúng một trong ba loại hoa: cúc, hồng, lan.
  2. Một ô vuông $1\times 1$ không có quá một chậu hoa.
  3. Với mỗi chậu hoa cho trước, số lượng chậu trồng hoa khác loài với nó trên cùng hàng ngang và số lượng chậu trồng hoa khác loài với nó trên cùng hàng dọc thì có tổng là $3$.

Hỏi người làm vườn có thể đặt được tối đa bao nhiêu chậu cây mà có đủ cả ba loại hoa trong vườn và thỏa mãn cả ba điều kiện trên?

 

Bài 3. Cho tam giác $ABC$ nhọn, không cân. Đường tròn nội tiếp tam giác $ABC$ tiếp xúc với các cạnh $BC,CA,AB$ theo thứ tự tại $D,E,F$. Gọi $X,Y,Z$ lần lượt là chân đường cao hạ từ đỉnh $A,B,C$ của tam giác $ABC$. Gọi $A'$ là điểm đối xứng với $X$ qua $EF$, gọi $B'$ là điểm đối xứng với $Y$ qua $FD$ và $C'$ là điểm đối xứng với $Z$ qua $DE$. Chứng minh rằng tam giác $ABC$ đồng dạng với tam giác $A'B'C'$.

 

 

Nguồn: Hướng tới Olympic Toán VN (nhóm facebook)




#744347 Mọi tập con khác rỗng bất kì của S đều là lũy thừa đúng.

Gửi bởi nhungvienkimcuong trong 24-03-2024 - 18:53

Chứng minh rằng với mỗi n $\in \mathbb{Z^{+}}$ đều tồn tại 1 tập S gồm n số nguyên dương sao cho tổng của mọi tập con khác rỗng bất kì của S đều là lũy thừa đúng.

Đề ở đây chắc muốn nói lũy thừa có số mũ lớn hơn $1$.

 

Đề bài khó chịu, để có thể hiệu chỉnh đồng thời $2^n-1$ tập hợp là một việc mới nghe là thấy rất khó khăn, thế nên cần nhìn nhận vấn đề một cách kĩ càng hơn. Giả dụ có sẵn họ tập hợp $\{S_i\}_{i=1}^{2^n-1}$, ta mong muốn từ đây xây dựng họ tập hợp mới sao cho tổng các phần tử trong mỗi tập hợp đều là lũy thừa đúng. Đơn giản nhất để thay đổi tổng các phần tử là

  • Cùng cộng các phần tử một lượng $a$, như vậy trở thành họ tập hợp mới là $\{S_i+a\}$,
  • Hoặc cùng nhân các phần tử một lượng $a$, trở thành họ tập hợp mới $\{a S_i\}$.

Trong hai cách này thì việc nhân phần tử sẽ dễ thực hiện hơn (nháp các trường hợp đơn giản là thấy; chẳng hạn $S_1=\{2\},S_2=\{3\},S_3=\{10\}$). Từ đây ta thấy rằng chỉ cần chứng minh kết quả sau:

Mệnh đề

Cho các tập hợp khác rỗng $S_1,S_2,\dots,S_k\subset \mathbb{N}^*$. Kí hiệu $s_i$ là tổng các phần tử của tập hợp $S_i$. Khi đó tồn tại số nguyên dương $a$ sao cho: với mỗi $i\in\{1,2,\dots,k\}$ thì tích $a\cdot s_i$ đều là lũy thừa có số mũ lớn hơn $1$.




#744342 Một hình vuông có độ dài cạnh là 12. Người ta tô màu tâm của các ô vuông đơn...

Gửi bởi nhungvienkimcuong trong 24-03-2024 - 12:52

 

Một hình vuông có độ dài cạnh là 12. Người ta tô màu tâm của các ô vuông đơn vị bởi màu xanh hoặc đỏ. Chứng minh có bốn điểm cùng màu là bốn đỉnh của một hình chữ nhật.

 

Bài này là một phiên bản khó hơn. Các bài có yêu cầu chứng minh bốn đỉnh của hình chữ nhật hoặc hình thang đều có cách xử lí tương tự như thế.




#744341 $\sum 3\sqrt{tan\frac{A}{2}.tan...

Gửi bởi nhungvienkimcuong trong 24-03-2024 - 12:13

Chứng minh rằng trong mọi tam giác $ABC$ ta có:
$3\sqrt{tan\frac{A}{2}.tan\frac{B}{2}}+4\sqrt{tan\frac{B}{2}.tan\frac{C}{2}}+5\sqrt{tan\frac{C}{2}.tan\frac{A}{2}}\leq 5\sqrt{2}$
 

Chú ý đẳng thức $\tan\frac{A}{2}\tan\frac{B}{2}+\tan\frac{B}{2}\tan\frac{C}{2}+\tan\frac{C}{2}\tan\frac{A}{2}=1$ là được.




#744340 Tìm hàm $f$ thoả $f\left ( x+y+f\left ( y \righ...

Gửi bởi nhungvienkimcuong trong 24-03-2024 - 11:59

Tìm tất cả các hàm số $f:\mathbb{R}\rightarrow \mathbb{R}$ thỏa mãn

$f\left ( 0 \right )=2024$ và $f\left ( x+y+f\left ( y \right ) \right )=f\left ( f\left ( x \right ) \right )+2023y,\forall x,y \in \mathbb{R}$

Ta sẽ chứng minh hàm số đơn ánh, xét hai số $a$ và $b$ thỏa mãn $f(a)=f(b)$ (gọi giá trị này là $c$). Lần lượt thay $x:=a,y:=b$ và $x:=b,y:=a$ vào giả thiết ta có

\[f(a+b+c)=f(c)+2023b\quad \text{và}\quad f(b+a+c)=f(c)+2023a.\]

So sánh hai đẳng thức này thu được $a=b$, nghĩa là hàm $f$ đơn ánh. Từ đây thay $y:=0$ vào giả thiết suy ra $f(x)=x+2024$.




#743800 $4\left ( a^{n}+1 \right )$ là lập phương của m...

Gửi bởi nhungvienkimcuong trong 23-02-2024 - 21:54

(Iran Second Round 2008). Tìm a nguyên dương sao cho $4\left ( a^{n}+1 \right )$ là lập phương của một số nguyên dương với mọi n.

Sau đây là lời giải của THPT.

 

Với mỗi số nguyên dương $n$ đặt $x_n=\sqrt[3]{4(a^n+1)}$, theo giả thiết thì $x_n\in \mathbb{Z}$. Ta có

\[x_{n+3}-ax_n=\sqrt[3]{4(a^{n+3}+1)}-a\sqrt[3]{4(a^n+1)}=\frac{4(1-a^3)}{x_{n+3}^2+x_{n+3}\cdot ax_n+(ax_n)^2}.\]

Vì $\lim_{n\to \infty} x_n=+\infty$ nên $\lim_{n\to \infty} (x_{n+3}-ax_n)=0$, tuy nhiên vì là dãy số nguyên nên tồn tại số nguyên dương $N$ sao cho

\[x_{n+3}=ax_n,\quad \forall n\ge N.\]

Từ đây tìm được $a=1$, thử lại thỏa mãn.

 

Ghi chú. Có thể tham khảo thêm phương pháp "sử dụng giải tích trong số học" thông qua tài liệu ở đây.




#743616 Number of odd divisors of $k$

Gửi bởi nhungvienkimcuong trong 16-02-2024 - 12:26

Chứng minh với mỗi số nguyên dương $n$ ta có đẳng thức sau:
\[\sum_{k\ge 1}\left\lfloor\dfrac{n}{2k-1}\right\rfloor =\sum_{k\ge 1}\left\lfloor\dfrac{n+k}{2k}\right\rfloor \]
Ý nghĩa: VT\eqref{e1} là tổng của: số các số chia hết cho $1,3,5,…$ không quá $n$

Chắc đây chỉ là bài toán khởi đầu thôi  :icon6: , vì hết sức đơn giản chỉ cần sử dụng đẳng thức Hermite

\[\left \lfloor \frac{n+k}{2k} \right \rfloor=\left \lfloor \frac{n}{k} \right \rfloor-\left \lfloor \frac{n}{2k} \right \rfloor\]

cho vế phải.




#743401 Chứng minh rằng dãy $(\lfloor x\rfloor)_{n\in\m...

Gửi bởi nhungvienkimcuong trong 08-02-2024 - 13:16

Cho $q$ là một số hữu tỷ không nguyên lớn hơn $1$. Xét dãy: $x_1=q, x_{n+1}=q\{x_n\}$. Chứng minh rằng dãy $(\lfloor x\rfloor)_{n\in\mathbb{N}}$ sẽ không bao giờ tuần hoàn, tức không tồn tại $N,k\neq 0$ sao cho $\lfloor x_{n+k}\rfloor=\lfloor x_n\rfloor,\forall n>N$.

Giả sử tồn tại số hai số nguyên dương $N$ và $k$ thỏa mãn $\lfloor x_{n+k}\rfloor=\lfloor x_n\rfloor$ với mọi số nguyên $n\ge N_1=N+1$. Vì $\lim_{n\to\infty}\frac{1}{q^n}=0$ nên tồn tại số nguyên dương $\alpha$ sao cho

\[|x_{N_1+k}-x_{N_1}|>\frac{1}{q^\alpha}.\tag{$\ast$}\]

Tiếp theo ta sẽ chỉ ra mâu thuẫn, bắt đầu với $\lfloor x_{N_1+\alpha+k}\rfloor=\lfloor x_{N_1+\alpha}\rfloor$ ta có

\[|x_{N_1+\alpha+k}-x_{N_1+\alpha}|<1\implies \Big|q\{x_{N_1+\alpha-1+k}\}-q\{x_{N_1+\alpha-1}\}\Big|<1.\]

Mặt khác $\lfloor x_{N_1+\alpha-1+k}\rfloor=\lfloor x_{N_1+\alpha-1}\rfloor$ dẫn đến

\[|x_{N_1+\alpha-1+k}-x_{N_1+\alpha-1}|=\Big|\{x_{N_1+\alpha-1+k}\}-\{x_{N_1+\alpha-1}\}\Big|<\frac{1}{q}.\]

Tiếp tục như thế ta có $|x_{N_1+k}-x_{N_1}|<\frac{1}{q^\alpha}$, mâu thuẫn với $(\ast)$.

 

P.s. Đọc lại thì thấy xét thiếu trường hợp  :mellow:




#743327 $a^2+d^2=b^2+c^2=P$

Gửi bởi nhungvienkimcuong trong 02-02-2024 - 14:20

Cho a, b, c, d là các số nguyên dương đôi một phân biệt thoả mãn $a^2+d^2=b^2+c^2=P$. Chứng minh rằng: 
a. $P$ là hợp số.
b. $ab+cd$ và $ac+bd$ không thể đồng thời là số nguyên tố.

Giả sử $a>b>c>d$.

 

----------------------------------------------------------------------------------------------------------------

Cả hai ý đều có cách xử lí tương tự nhau bằng cách để ý rằng

\[(ab+cd)(ac+bd)=ad(b^2+c^2)+bc(a^2+d^2)=(ad+bc)P.\]

Như vậy nếu $P$ là số nguyên tố (phản chứng ý a) hoặc cả $ab+cd,ac+bd$ đều là số nguyên tố (phản chứng ý b) thì ta đều suy ra được

\[P\le ab+cd\quad\text{hoặc}\quad P\le ac+bd.\]

Không mất tính tổng quát giả sử $P\le ab+cd$, mặt khác

\[P^2=(a^2+d^2)(b^2+c^2)\ge (ab+cd)^2.\]

Như vậy dấu bằng của bất đẳng thức trên xảy ra nên $\frac{a}{b}=\frac{d}{c}$, tuy nhiên điều này mâu thuẫn với $\frac{a}{b}>1>\frac{d}{c}$.

 

----------------------------------------------------------------------------------------------------------------

Ngoài ra ý a còn có một cách xử lí khác thông qua kết quả sau. 

Bổ đề

Với $A,B,C,D$ là các số nguyên dương thỏa mãn $AB=CD$, khi đó tồn tại các số nguyên dương $x,y,z,t$ sao cho

\[A=xy,\ B=zt\quad\text{và}\quad C=xz,\ D=yt.\]

Từ giả thiết có được $(a-b)(a+b)=(c-d)(c+d)$. Áp dụng Theorem suy ra tồn tại các số nguyên dương $x,y,z,t$ thỏa mãn

\[a-b=xy,\ a+b=zt\quad\text{và}\quad c-d=xz,\ c+d=yt.\]

Từ đây thu được $a=\frac{xy+zt}{2}$ và $d=\frac{yt-xz}{2}$, dẫn đến

\[P=\frac{(xy+zt)^2+(yt-xz)^2}{4}=\frac{(x^2+t^2)(y^2+z^2)}{4}.\]

Từ đẳng thức này dễ dàng chứng minh $P$ là hợp số.

 

 

Ghi chú. Có thể xử lí bài 6 IMO 2001 tương tự như trên.




#743114 Min max của $\frac{b+c}{a}+\frac{c+a}{b}+\frac{a+b}{c}$

Gửi bởi nhungvienkimcuong trong 19-01-2024 - 10:08

Cho $a,b,c$ là các số nguyên dương đôi một phân biệt và thỏa mãn: $a+b+c=16$. Tìm GTNN và GTLN của

$$P=\frac{b+c}{a}+\frac{c+a}{b}+\frac{a+b}{c}$$

P/s: Bài này em chế lại từ đề HSG toán 9 Hà Nội năm ngoái mà giờ khó quá!!!! Đăng lên cho mọi người thảo luận ạ.

Dù điều kiện của đề là các số đôi một phân biệt nhưng thật ra cách xử lí cũng tương tự ở đây.

 

Vì các biến là số nguyên nên cách xử lí hơi khác, tuy nhiên dấu bằng xảy ra thì khá tương tự khi các biến là số thực. Cụ thể dấu bằng trong trường hợp 

  • GTNN khi các biến "khá" bằng nhau (hơn kém nhau $1$ đơn vị).
  • GTLN đạt tại biên, nghĩa là một biến bằng $1$ và một biến bằng $2$.



#743104 Số hoán vị không có số i nào đứng ở vị trí thứ i

Gửi bởi nhungvienkimcuong trong 18-01-2024 - 21:03

Có bao nhiêu hoán vị của tập S={1,2,3,4,5,6} trong đó không có số i nào đứng ở vị trí thứ i với i chạy từ 1 đến 6

Với mỗi số nguyên dương $n$, gọi $F_n$ là số song ánh $f\colon \{1,2,\dots,n\}\to \{1,2,\dots,n\}$ sao cho không có điểm bất động (nghĩa là $f(i)\neq i$ với mọi $i$). Ta có kết quả

\[F_n=n!\left ( \frac{1}{2!}-\frac{1}{3!}+\dots+\frac{(-1)^n}{n!} \right ).\]

Bên cạnh sử dụng nguyên lí bù trừ, còn có thể chứng minh thông qua dãy truy hồi bằng cách chứng tỏ $F_n=(n-1)(F_{n-1}+F_{n-2})$.




#743018 Chứng minh rằng với bất kỳ số tự nhiên n nào thì $1978^{n} – 1...

Gửi bởi nhungvienkimcuong trong 15-01-2024 - 20:04

[Đề thi vô địch toàn Liên Xô  – lớp 8, năm 1978]

Chứng minh rằng với bất kỳ số tự nhiên n nào thì $1978^{n} – 1$ không thể  chia hết cho  $1000^{n} – 1$.

Bài dành cho lớp nhỏ cũng như đã khá lâu nên chắc hẳn chỉ cần những kiến thức cơ bản nhất.

 

Giả sử tồn tại số tự nhiên $n$ sao cho $1000^n-1$ là ước của $1978^n-1$. Như vậy $1000^n-1$ cũng là ước của

\[(1978^n-1)-(1000^n-1)=2^n(989^n-500^n).\]

Vì $1000^n-1$ lẻ nên

\[1000^n-1\mid 989^n-500^n\implies 1000^n-1\le 989^n-500^n.\]

Từ đây ta có mâu thuẫn.